1989 IMO Problems/Problem 5

Revision as of 01:55, 24 August 2024 by Sansgankrsngupta (talk | contribs)
(diff) ← Older revision | Latest revision (diff) | Newer revision → (diff)

Problem

Prove that for each positive integer $n$ there exist $n$ consecutive positive integers none of which is an integral power of a prime number.

Solution 1

There are at most $1+\sqrt[2]{n}+\sqrt[3]{n}+\sqrt[4]{n}+...+\sqrt[\left\lfloor \log_2(n)\right\rfloor]{n} \leq 1+ \sqrt n log_2(n)$ 'true' powers $m^k , k\geq 2$ in the set $\{1,2,...,n\}$. So when $p(n)$ gives the amount of 'true' powers $\leq n$ we get that $\lim_{n \to \infty} \frac{p(n)}{n} = 0$.

Since also $\lim_{n \to \infty} \frac{\pi(n)}{n} = 0$, we get that $\lim_{n \to \infty} \frac{p(n)+\pi(n)}{n} = 0$. Now assume that there is no 'gap' of lenght at least $k$ into the set of 'true' powers and the primes. Then this would give that $\frac{p(n)+\pi(n)}{n} \geq \frac{1}{k}$ for all $n$ in contrary to the above (at east this proves a bit more).

Edit: to elementarize the $\lim_{n \to \infty} \frac{\pi(n)}{n} = 0$ part: Look $\mod (k+1)!$. Then all numbers in the residue classes $2,3,4,...,k+1$ are not primes (except the smallest representants sometimes). So when there wouldn't exist a gap of length $k$, there has to be a 'true' power in each of these gaps of the prime numbers, so at least one power each $(k+1)!$ numbers, again contradicting $\lim_{n \to \infty} \frac{p(n)}{n} = 0$.

This solution was posted and copyrighted by ZetaX. The original thread for this problem can be found here: [1]

Solution 2

By the Chinese Remainder Theorem, there exists $x$ such that: $x \equiv -1\;mod\;p_1 q_1\\  x \equiv -2\;mod\;p_2 q_2\\ x \equiv -3\;mod\;p_3 q_3\\ ...\newline x \equiv -n\;mod\;p_n q_n$ where $p_1, p_2, ..., p_n, q_1, q_2, ..., q_n$ are distinct primes. The $n$ consecutive numbers $x+1, x+2, ..., x+n$ each have at least two prime factors, so none of them can be expressed as an integral power of a prime.

Solution 3

OG, If $n=1$, then select any composite number $Claim:$ If $n>1$, then the consectutive number $(2n+3)!+2, (2n+3)!+3 \cdots (2n+3)!+(n+1)$, give the desired $n$ consecutive numbers as each of the numbers are divisible by at least 2 primes. Proof: Each of the numbers are $(2n+3)!+k, 2 \leq k \leq n+1$, Now since $2k \leq (2n+3)! \implies k^2|(2n+3)$, Thus $(2n+3)!+k= k(kq+1)$, which clearly has atleast 2 prime divisors. Hence DONE, OG


See Also

1989 IMO (Problems) • Resources
Preceded by
Problem 4
1 2 3 4 5 6 Followed by
Problem 6
All IMO Problems and Solutions